Find a permutation $sigma$ maximizing the sum $sum_{i=1}^n {a_i over sigma(i)}$.












0












$begingroup$



Given $$a_1< a_2<dots < a_n,$$



find a permutation $sigma$ maximizing the sum >$$sum_{i=1}^n {a_i over sigma(i)}.$$




I can't figure our where to begin. I know that the solution is $sigma=e$, but I cannot prove it.










share|cite|improve this question











$endgroup$








  • 2




    $begingroup$
    Maybe worth trying: without loss of generality, assume the $a_i$'s are non-decreasing, and use the rearrangement inequality?
    $endgroup$
    – Clement C.
    Jul 16 '16 at 14:48






  • 1




    $begingroup$
    After the edit, and to provide more details to my previous comment: en.m.wikipedia.org/wiki/Rearrangement_inequality
    $endgroup$
    – Clement C.
    Jul 16 '16 at 14:56








  • 3




    $begingroup$
    For the answer to be $sigma=e$, I think you need $a_1>a_2>cdots>a_n$.
    $endgroup$
    – Thomas Andrews
    Jul 16 '16 at 14:57










  • $begingroup$
    Sorry for the lack of information. I edited it.
    $endgroup$
    – Razvan Paraschiv
    Jul 16 '16 at 14:59






  • 2




    $begingroup$
    Imagine $a_n$ is huge compared to the other terms. Then the maximum sum will certainly have $sigma(n)=1$
    $endgroup$
    – Joffan
    Jul 16 '16 at 15:08
















0












$begingroup$



Given $$a_1< a_2<dots < a_n,$$



find a permutation $sigma$ maximizing the sum >$$sum_{i=1}^n {a_i over sigma(i)}.$$




I can't figure our where to begin. I know that the solution is $sigma=e$, but I cannot prove it.










share|cite|improve this question











$endgroup$








  • 2




    $begingroup$
    Maybe worth trying: without loss of generality, assume the $a_i$'s are non-decreasing, and use the rearrangement inequality?
    $endgroup$
    – Clement C.
    Jul 16 '16 at 14:48






  • 1




    $begingroup$
    After the edit, and to provide more details to my previous comment: en.m.wikipedia.org/wiki/Rearrangement_inequality
    $endgroup$
    – Clement C.
    Jul 16 '16 at 14:56








  • 3




    $begingroup$
    For the answer to be $sigma=e$, I think you need $a_1>a_2>cdots>a_n$.
    $endgroup$
    – Thomas Andrews
    Jul 16 '16 at 14:57










  • $begingroup$
    Sorry for the lack of information. I edited it.
    $endgroup$
    – Razvan Paraschiv
    Jul 16 '16 at 14:59






  • 2




    $begingroup$
    Imagine $a_n$ is huge compared to the other terms. Then the maximum sum will certainly have $sigma(n)=1$
    $endgroup$
    – Joffan
    Jul 16 '16 at 15:08














0












0








0





$begingroup$



Given $$a_1< a_2<dots < a_n,$$



find a permutation $sigma$ maximizing the sum >$$sum_{i=1}^n {a_i over sigma(i)}.$$




I can't figure our where to begin. I know that the solution is $sigma=e$, but I cannot prove it.










share|cite|improve this question











$endgroup$





Given $$a_1< a_2<dots < a_n,$$



find a permutation $sigma$ maximizing the sum >$$sum_{i=1}^n {a_i over sigma(i)}.$$




I can't figure our where to begin. I know that the solution is $sigma=e$, but I cannot prove it.







real-analysis sequences-and-series permutations






share|cite|improve this question















share|cite|improve this question













share|cite|improve this question




share|cite|improve this question








edited Dec 22 '18 at 12:19









Shaun

9,366113684




9,366113684










asked Jul 16 '16 at 14:43









Razvan ParaschivRazvan Paraschiv

861615




861615








  • 2




    $begingroup$
    Maybe worth trying: without loss of generality, assume the $a_i$'s are non-decreasing, and use the rearrangement inequality?
    $endgroup$
    – Clement C.
    Jul 16 '16 at 14:48






  • 1




    $begingroup$
    After the edit, and to provide more details to my previous comment: en.m.wikipedia.org/wiki/Rearrangement_inequality
    $endgroup$
    – Clement C.
    Jul 16 '16 at 14:56








  • 3




    $begingroup$
    For the answer to be $sigma=e$, I think you need $a_1>a_2>cdots>a_n$.
    $endgroup$
    – Thomas Andrews
    Jul 16 '16 at 14:57










  • $begingroup$
    Sorry for the lack of information. I edited it.
    $endgroup$
    – Razvan Paraschiv
    Jul 16 '16 at 14:59






  • 2




    $begingroup$
    Imagine $a_n$ is huge compared to the other terms. Then the maximum sum will certainly have $sigma(n)=1$
    $endgroup$
    – Joffan
    Jul 16 '16 at 15:08














  • 2




    $begingroup$
    Maybe worth trying: without loss of generality, assume the $a_i$'s are non-decreasing, and use the rearrangement inequality?
    $endgroup$
    – Clement C.
    Jul 16 '16 at 14:48






  • 1




    $begingroup$
    After the edit, and to provide more details to my previous comment: en.m.wikipedia.org/wiki/Rearrangement_inequality
    $endgroup$
    – Clement C.
    Jul 16 '16 at 14:56








  • 3




    $begingroup$
    For the answer to be $sigma=e$, I think you need $a_1>a_2>cdots>a_n$.
    $endgroup$
    – Thomas Andrews
    Jul 16 '16 at 14:57










  • $begingroup$
    Sorry for the lack of information. I edited it.
    $endgroup$
    – Razvan Paraschiv
    Jul 16 '16 at 14:59






  • 2




    $begingroup$
    Imagine $a_n$ is huge compared to the other terms. Then the maximum sum will certainly have $sigma(n)=1$
    $endgroup$
    – Joffan
    Jul 16 '16 at 15:08








2




2




$begingroup$
Maybe worth trying: without loss of generality, assume the $a_i$'s are non-decreasing, and use the rearrangement inequality?
$endgroup$
– Clement C.
Jul 16 '16 at 14:48




$begingroup$
Maybe worth trying: without loss of generality, assume the $a_i$'s are non-decreasing, and use the rearrangement inequality?
$endgroup$
– Clement C.
Jul 16 '16 at 14:48




1




1




$begingroup$
After the edit, and to provide more details to my previous comment: en.m.wikipedia.org/wiki/Rearrangement_inequality
$endgroup$
– Clement C.
Jul 16 '16 at 14:56






$begingroup$
After the edit, and to provide more details to my previous comment: en.m.wikipedia.org/wiki/Rearrangement_inequality
$endgroup$
– Clement C.
Jul 16 '16 at 14:56






3




3




$begingroup$
For the answer to be $sigma=e$, I think you need $a_1>a_2>cdots>a_n$.
$endgroup$
– Thomas Andrews
Jul 16 '16 at 14:57




$begingroup$
For the answer to be $sigma=e$, I think you need $a_1>a_2>cdots>a_n$.
$endgroup$
– Thomas Andrews
Jul 16 '16 at 14:57












$begingroup$
Sorry for the lack of information. I edited it.
$endgroup$
– Razvan Paraschiv
Jul 16 '16 at 14:59




$begingroup$
Sorry for the lack of information. I edited it.
$endgroup$
– Razvan Paraschiv
Jul 16 '16 at 14:59




2




2




$begingroup$
Imagine $a_n$ is huge compared to the other terms. Then the maximum sum will certainly have $sigma(n)=1$
$endgroup$
– Joffan
Jul 16 '16 at 15:08




$begingroup$
Imagine $a_n$ is huge compared to the other terms. Then the maximum sum will certainly have $sigma(n)=1$
$endgroup$
– Joffan
Jul 16 '16 at 15:08










0






active

oldest

votes











Your Answer





StackExchange.ifUsing("editor", function () {
return StackExchange.using("mathjaxEditing", function () {
StackExchange.MarkdownEditor.creationCallbacks.add(function (editor, postfix) {
StackExchange.mathjaxEditing.prepareWmdForMathJax(editor, postfix, [["$", "$"], ["\\(","\\)"]]);
});
});
}, "mathjax-editing");

StackExchange.ready(function() {
var channelOptions = {
tags: "".split(" "),
id: "69"
};
initTagRenderer("".split(" "), "".split(" "), channelOptions);

StackExchange.using("externalEditor", function() {
// Have to fire editor after snippets, if snippets enabled
if (StackExchange.settings.snippets.snippetsEnabled) {
StackExchange.using("snippets", function() {
createEditor();
});
}
else {
createEditor();
}
});

function createEditor() {
StackExchange.prepareEditor({
heartbeatType: 'answer',
autoActivateHeartbeat: false,
convertImagesToLinks: true,
noModals: true,
showLowRepImageUploadWarning: true,
reputationToPostImages: 10,
bindNavPrevention: true,
postfix: "",
imageUploader: {
brandingHtml: "Powered by u003ca class="icon-imgur-white" href="https://imgur.com/"u003eu003c/au003e",
contentPolicyHtml: "User contributions licensed under u003ca href="https://creativecommons.org/licenses/by-sa/3.0/"u003ecc by-sa 3.0 with attribution requiredu003c/au003e u003ca href="https://stackoverflow.com/legal/content-policy"u003e(content policy)u003c/au003e",
allowUrls: true
},
noCode: true, onDemand: true,
discardSelector: ".discard-answer"
,immediatelyShowMarkdownHelp:true
});


}
});














draft saved

draft discarded


















StackExchange.ready(
function () {
StackExchange.openid.initPostLogin('.new-post-login', 'https%3a%2f%2fmath.stackexchange.com%2fquestions%2f1861299%2ffind-a-permutation-sigma-maximizing-the-sum-sum-i-1n-a-i-over-sigmai%23new-answer', 'question_page');
}
);

Post as a guest















Required, but never shown

























0






active

oldest

votes








0






active

oldest

votes









active

oldest

votes






active

oldest

votes
















draft saved

draft discarded




















































Thanks for contributing an answer to Mathematics Stack Exchange!


  • Please be sure to answer the question. Provide details and share your research!

But avoid



  • Asking for help, clarification, or responding to other answers.

  • Making statements based on opinion; back them up with references or personal experience.


Use MathJax to format equations. MathJax reference.


To learn more, see our tips on writing great answers.




draft saved


draft discarded














StackExchange.ready(
function () {
StackExchange.openid.initPostLogin('.new-post-login', 'https%3a%2f%2fmath.stackexchange.com%2fquestions%2f1861299%2ffind-a-permutation-sigma-maximizing-the-sum-sum-i-1n-a-i-over-sigmai%23new-answer', 'question_page');
}
);

Post as a guest















Required, but never shown





















































Required, but never shown














Required, but never shown












Required, but never shown







Required, but never shown

































Required, but never shown














Required, but never shown












Required, but never shown







Required, but never shown







Popular posts from this blog

To store a contact into the json file from server.js file using a class in NodeJS

Redirect URL with Chrome Remote Debugging Android Devices

Dieringhausen